Q10

User avatar
 
ManhattanPrepLSAT1
Thanks Received: 1909
Atticus Finch
Atticus Finch
 
Posts: 2851
Joined: October 07th, 2009
 
This post thanked 1 time.
 
 

Q10

by ManhattanPrepLSAT1 Sun Sep 25, 2016 1:49 am

Image
 
Rendona001
Thanks Received: 0
Vinny Gambini
Vinny Gambini
 
Posts: 8
Joined: September 27th, 2016
 
 
 

Re: Q10

by Rendona001 Fri Dec 02, 2016 1:11 am

Hi,

Can someone clarify what this question means? Or how to attack this type of question? Thanks!
User avatar
 
ohthatpatrick
Thanks Received: 3807
Atticus Finch
Atticus Finch
 
Posts: 4661
Joined: April 01st, 2011
 
This post thanked 8 times.
 
 

Re: Q10

by ohthatpatrick Fri Dec 02, 2016 2:12 pm

We call these nasty ones Rule Equivalency.

Over the last 20 tests, there has been one of these per Games sections. They're sometimes fine and sometimes miserable. I'm comfortable skipping them and saving them for last if nothing jumps out on the first pass.

The question stem is just asking, "Which of these answer choices could we put in place of Rule 2 and have the exact same game?"

Answer choices will be wrong because they would change the game in some way, specifically ...

1. Too Restrictive (they prevent something that used to be allowed)
or
2. Too Permissive (they allow something that used to be forbidden)


I start with Too Restrictive thinking, because you can use previous work to analyze that.

For (A), for example, it's saying that every time F is 7, H is 2. Check your previous scenarios -- if you have any scenarios where F was 7 but H was not 2, then you can eliminate this answer.

If an answer survives the Too Restrictive filter, then I ask myself, "Would this do the work of the rule being replaced?"

In this case, examining (A), would this rule force H to be before K and prevent H from being 1?

Doesn't look like it! If F is 7, then H can't be 1, and H would come sometime before the GK chunk.
But ... if F isn't 7, then all bets are off. Nothing would prevent H from being 1st or from H coming after K.

So, in summary, for each answer choice, ask yourself:
1. Can I find a previous scenario that breaks this rule? (that shows the rule is too restrictive)
and
2. Does this do the work of the rule being replaced? or, alternatively, could I follow THIS rule but break the ORIGINAL rule? (that shows the rule is too permissive)
 
hnadgauda
Thanks Received: 12
Elle Woods
Elle Woods
 
Posts: 77
Joined: March 31st, 2017
 
 
 

Re: Q10

by hnadgauda Thu Apr 27, 2017 4:28 pm

I found the wording of answer choice E to be confusing. How do you diagram this rule? I came up with: F--H--K
 
JesseKirkland
Thanks Received: 1
Vinny Gambini
Vinny Gambini
 
Posts: 2
Joined: March 19th, 2017
 
This post thanked 1 time.
 
 

Re: Q10

by JesseKirkland Sat Jun 24, 2017 9:05 pm

The F - H - K graphing of answer choice (E) looks good to me. I was reading the word "which" in the answer choice as referring to H. So K is after H and H is after F.
 
e.sterlingsmith
Thanks Received: 1
Vinny Gambini
Vinny Gambini
 
Posts: 12
Joined: March 11th, 2013
 
 
 

Re: Q10

by e.sterlingsmith Fri Jun 30, 2017 5:30 pm

I also thought E worked. F diagrammed it F- H-GK which would stop H from being in 1 and force it ahead of GK
User avatar
 
ohthatpatrick
Thanks Received: 3807
Atticus Finch
Atticus Finch
 
Posts: 4661
Joined: April 01st, 2011
 
This post thanked 1 time.
 
 

Re: Q10

by ohthatpatrick Wed Jul 19, 2017 10:12 pm

(E) definitely "does the work" of the rule we're getting rid of, since F - H - GK guarantees that H is before K but not 1st.

However, F - H - GK does not give us a game identical to the game we were playing up until that question, because in the actual game, F can be 1 or 7.

According to this new rule, F could never be 7. Thus, it does not have the same effect (it does not produce an identical game)
 
RodrigoA220
Thanks Received: 1
Vinny Gambini
Vinny Gambini
 
Posts: 6
Joined: October 22nd, 2017
 
 
 

Re: Q10

by RodrigoA220 Fri Nov 10, 2017 11:38 am

Can someone walk me through why answer choice D DOES have the same effect as rule 2? I understand attacking these is long and tedious, but I'm having trouble conceptualizing how this rule replaces rule 2
User avatar
 
ohthatpatrick
Thanks Received: 3807
Atticus Finch
Atticus Finch
 
Posts: 4661
Joined: April 01st, 2011
 
This post thanked 4 times.
 
 

Re: Q10

by ohthatpatrick Sat Nov 18, 2017 2:16 am

This question is an outlier, when it comes to Rule Equivalency.

It is by far the hardest one I know of, and you never could have predicted the mechanism the correct answer uses.

(D) Unless Hibiscus is second, it must be somewhere between Magnolia and Gardenias.

If H isn't 2nd, then it's somewhere between M and G.

We need to achieve the same effect as
H is before K, but H can't be first.


Does the rule in (D) achieve the effect that "H can't be first"?

The trigger is "If H isn't 2nd".

If H is 2nd, then it obviously can't be first.
and according to the rule,
If H isn't 2nd, it's between M and G, so it can't be first.

That part checks out. Does this rule force H to be earlier than K?

If H is 2nd, then H is before K, since the GK chunk would have to come later.

If H isn't 2nd, the rule in (D) says that H is between M and G,
so it's either
GK - H - M
or
M - H - GK.

In the M - H - GK possibility, H is before K.

The GK - H - M possibility is actually NOT a possibility, because it violates the rule that M has to be in the first three spots.

Let me reiterate that THIS ANSWER CHOICE IS A UNIQUE FLAVOR OF MISERY.
 
RodrigoA220
Thanks Received: 1
Vinny Gambini
Vinny Gambini
 
Posts: 6
Joined: October 22nd, 2017
 
 
 

Re: Q10

by RodrigoA220 Mon Nov 20, 2017 3:35 pm

that makes sense, now, thank you!

I won't stress too much knowing it's that difficult of a question, but understanding it feels helpful.
 
emilysem
Thanks Received: 0
Vinny Gambini
Vinny Gambini
 
Posts: 2
Joined: January 29th, 2018
 
 
 

Re: Q10

by emilysem Sun Feb 11, 2018 5:18 pm

So this is the only question out of the set that I have a super hard time understanding, even after reading the above explanations. Is this question type relatively uncommon (AKA, can I get away with not knowing how to do this because it's stressing me out)?
User avatar
 
ohthatpatrick
Thanks Received: 3807
Atticus Finch
Atticus Finch
 
Posts: 4661
Joined: April 01st, 2011
 
 
 

Re: Q10

by ohthatpatrick Thu Feb 15, 2018 2:05 pm

You can/should be thinking that Equivalent Rule questions are usually harder than an average Games question, so it's sensible to think that we might live with eating that missed point, if it saves us time and allows us to get more attainable points later in the section.

I am very willing to circle one of those and come back (kind of like long Matching questions in Logical Reasoning ... I'd rather save them until I've done all the other questions in the section).

BUT ....

Not sure if you were registering this as you read the thread:
THIS PARTICULAR Equivalent Rule question is BY FAR the meanest ever. Many correct answers to Equivalent Rule are WAY EASIER than this is. So you can definitely have a negative prejudice against Equivalent Rule, but it can't be on the basis of THIS question, which is a total outlier.
 
AnnaC659
Thanks Received: 3
Jackie Chiles
Jackie Chiles
 
Posts: 40
Joined: January 03rd, 2018
 
 
 

Re: Q10

by AnnaC659 Sat May 19, 2018 1:10 am

Can someone go through C for me?

Thank you!
User avatar
 
ohthatpatrick
Thanks Received: 3807
Atticus Finch
Atticus Finch
 
Posts: 4661
Joined: April 01st, 2011
 
 
 

Re: Q10

by ohthatpatrick Mon May 21, 2018 1:05 pm

C is saying that H will always be in spots 2, 3, 4, or 5.

So we're asking ourselves 2 questions:
1. Was that always true before (or is this too restrictive)?

2. Following this new rule, can I break the old rule (is this new rule too permissive)?


#1 is true. Since H couldn't be 1, and we had a H - GK rule, H was always going to be limited to 2, 3, 4, or 5.

So onto #2. Following this rule, is it possible to break the original rule? (i.e. is it possible to get K before H)

Looks like it. I just tried this scenario. It honors all the original rules (other than rule 2)
F M G K H I L
and it honors choice (C).

So (C) is too permissive. It's not a rule that results in the same game, because (C) would allow for a scenario like
F M G K H I L
whereas the rule 2 would never allow for that.

Hope this helps.
 
shanks
Thanks Received: 0
Vinny Gambini
Vinny Gambini
 
Posts: 1
Joined: September 30th, 2018
 
 
 

Re: Q10

by shanks Sun Oct 14, 2018 5:47 pm

I'm on the boat that these are the worst questions ever, but for some reason I raced through this one without significant issue.

First pass... I reviewed previous work/questions
(A) and (B) were quickly eliminated because Q6 answer (C) shows that both are possible.

Second pass... I focused on my rules
(C) was eliminated because it didn't force Hibiscus before the [GK] chunk. (If H was 6, where does the chunk go?...BEFORE H)
(D) was eliminated because new rule F-H-K, eliminated possibility of F going 7th. (Too Restrictive).

Third...if time allows test (D)
Start with H is not 2nd.
If H3, then _ _ H_ _ _ _, F1, M2, [GK]4-7, (IL)4-7 = FMH(GK,IL)
If H4 then _ I/L _ H_ _ _, M1or3, [GK] 5-7, F7 = MILHGKF, or ILMHGKF
If H5 then F M (I,L) H G K,
*If H is 2nd, then _ H _ _ _ _ _, M1or3, [GK]2-7, (IL)3-7, F1or7 = FHM(GK,IL), or MH(GK,IL)F
All rules appear to work, nothing new is added to game, nothing is taken away. I am confident in (D).
 
PaulW588
Thanks Received: 0
Vinny Gambini
Vinny Gambini
 
Posts: 1
Joined: October 19th, 2018
 
 
 

Re: Q10

by PaulW588 Wed Nov 07, 2018 8:54 pm

My anxiety with these questions is I freeze and don't know what to really do to get to the answer. Normally with a confusing LG you just plug away, but these types of questions just get me frazzled. Would someone explain what my process should be with this, on a detailed level? I really hate this question type.
User avatar
 
ohthatpatrick
Thanks Received: 3807
Atticus Finch
Atticus Finch
 
Posts: 4661
Joined: April 01st, 2011
 
 
 

Re: Q10

by ohthatpatrick Thu Nov 08, 2018 3:26 pm

Correct answers have to satisfy two things:
1. They allow everything that was allowed before
2. They forbid everything that was forbidden before

You can choose to evaluate all the answers for #1 and then circle back to #2,
or you can evaluate each answer as you go for both, or you can decide on the fly (i.e. PLAN to check out all the answers for #1, but maybe you get a strong intuitive sense that you might be looking at the right answer and decide to just go ahead with checking #2 while you're there)


To check #1, you simply ask:
- Was this true before? If not, eliminate. If so, keep.
- Do I have any previous work that goes against this new rule? If so, eliminate.

To check #2, you ask:
- Does this new rule do the work of the old rule? If not, eliminate. If so, pick it.

- Can I write a scenario that obeys the new rule but breaks the old rule? If so, eliminate.


Put more simply:
1st - eliminate any answers that go against any of your previous work or that just sound to you like they're adding restrictions you don't remember having before

2nd - convince yourself that the new rule forces the same reality as the old rule, or prove the answer wrong by writing a scenario that obeys the new rule while breaking the old rule


Hope this helps.
 
DevorahK415
Thanks Received: 0
Vinny Gambini
Vinny Gambini
 
Posts: 10
Joined: December 01st, 2019
 
 
 

Re: Q10

by DevorahK415 Sat May 02, 2020 8:44 pm

A few questions:

1. Patrick wrote:
"The GK - H - M possibility is actually NOT a possibility, because it violates the rule that M has to be in the first three spots".
ISN'T IT BECAUSE IT ALSO VIOLATES THE H-GK RULE? WHY DO WE HAVE TO BRING IN M TO SHOW THAT H MUST BE SECOND OTHERWISE IT WHEN IT IS IN BETWEEN M & G IT WILL VIOLATE THE H-K RULE? FEELS LIKE MORE MISERY :-)"

2. Can you please go through B? I'm guessing that it is mixing the first and second original rules, as well as parts of the fifth and sixth rule. Is that the problem (to understand why it is wrong?

3. I think when I first read this question, I thought I should be testing the answers. Not so. What we should be doing is taking the answers as fact and then testing those against not only the rule that the answer is meant to replace and then the other original rules. If this is correct, how exactly are we supposed to do that quickly?

Thanks!
 
Laura Damone
Thanks Received: 94
Atticus Finch
Atticus Finch
 
Posts: 468
Joined: February 17th, 2011
 
 
 

Re: Q10

by Laura Damone Mon May 18, 2020 6:54 pm

Hi!

The reason we need to bring M into the equation is because we're testing D to see if it can replace the HK rule. In other words, does D force compliance with the HK rule in some other way. If GK - H - M was possible, it wouldn't force H to be before K. That would make the answer incorrect. But because M must be 1-3, GK - H - M is impossible, so putting H between M and G does mean H precedes K. Misery, indeed!

B is wrong because it forces F or M to be first in all cases. But in the original game, F or M didn't have to be first. I or L could also be first. That means B would have a different effect on the game than the rule it is replacing.

Regarding a general approach: unless you want a perfect game score, there's no shame in skipping these. They're often the hardest question on the section, so if you aren't already finishing on time, these are good ones to axe. But, if you do attempt them, on quick and dirty way is to use prior work. If an answer would force you to change an existing hypothetical to comply, it's the wrong answer. Any answer that isn't represented in every hypo is wrong. Eliminate those and check out the contenders. Do they force compliance with the rule you're replacing? And do they do it without invoking any additional rules? If yes, you've got the right answer. If no, keep looking.

Hope this helps!
Laura Damone
LSAT Content & Curriculum Lead | Manhattan Prep
 
AnnaT620
Thanks Received: 0
Elle Woods
Elle Woods
 
Posts: 51
Joined: May 25th, 2020
 
 
 

Re: Q10

by AnnaT620 Wed Jul 22, 2020 12:10 pm

Hi Patrick

This question really tripped me up - what would a Rule Equivalency question usually look like? You mentioned that this is the hardest one you've seen so far?

The approach for this one almost seemed like it could be a bit of a time trap? Would you skip this and come back to it - or is the process for these types of questions faster after a while?

Thanks!
Anna